HESI Exam 6 2024-26

16 July 2024

HESI Exam 6 2024-26

61. In a long term rehabilitation care unit a client with spinal cord injury complains of a pounding headache. The client is sitting in a wheelchair watching television in the assigned room. Further assessment by the nurse reveals excessive sweating, a splotchy rash, pilomotor erection, facial flushing, congested nasal passages and a heart rate of 50. The nurse should do which action next?

A)  Take the client’s respirations, blood pressure (BP), temperature and then pupillary responses

B)  Place the client into the bed and administer the ordered PRN analgesic

C)  Check the client for bladder distention and the client’s urinary catheter for kinks

D)  Turn the television off and then assist client to use relaxation techniques

The correct answer is C: Check the client for bladder distention and the client'’s urinary catheter for kinks

 

 

62. The nurse is performing a physical assessment on a client with insulin dependent diabetes mellitus. Which client complaint calls for immediate nursing action?

A)  Diaphoresis and shakiness

B)  Reduced lower leg sensation

C)  Intense thirst and hunger

D)  Painful hematoma on thigh

The correct answer is A: Diaphoresis and shakiness

 

 

63. The nurse is teaching a client about the healthy use of ego defense mechanisms. An appropriate goal for this client would be

A)  Reduce fear and protect self-esteem

B)  Minimize anxiety and delay apprehension

C)  Avoid conflict and leave unpleasant situations

D)  Increase independence and communicate more often

The correct answer is A: Reduce fear and protect self-esteem

 

64 In reviewing the assessment data of a client suspected of having diabetes insipidus, the nurse expects which of the following after a water deprivation test? A) Increased edema and weight gain

B)  Unchanged urine specific gravity

C)  Rapid protein excretion

D)  Decreased blood potassium

The correct answer is B: Unchanged urine specific gravity

 

 

65. The nurse is evaluating the growth and development of a toddler with AIDS. The nurse would anticipate finding that the child has

A)  Achieved developmental milestones at an erratic rate

B)  Delay in musculoskeletal development

C)  Displayed difficulty with speech development

D)  Delay in achievement of most developmental milestones

The correct answer is D: Delayed in achieving all developmental milestones

66. A client was admitted with a diagnosis of pneumonia. When auscultating the client’s breath sounds, the nurse hears inspiratory crackles in the right base. Temperature is 102.3 degrees Fahrenheit orally. What finding would the nurse expect? A) Flushed skin

B)  Bradycardia

C)  Mental confusion

D)  Hypotension

The correct answer is C: Mental confusion

 

 

67. Postoperative orders for a client undergoing a mitral valve replacement include monitoring pulmonary artery pressure togetherwith pulmonary capillary wedge pressure with a pulmonary artery

catheter. This action by the nurse will assess A) Right ventricular pressure

B)  Left ventricular end-diastolic pressure

C)  Acid-Base balance

D)  Coronary artery stability

The correct answer is B: Left ventricular end diastolic pressure

 

 

68. The nurse is providing instructions for a client with asthma who is sensitive to house dust-mites. Which information about prevention of asthma episodes would be the most helpful to include during the teaching?

A)  Change the pillow covers every month

B)  Wash bed linens in warm water with a cold rinse

C)  Wash and rinse the bed linens in hot water D) Use air filters in the furnace system

The correct answer is C: Wash and rinse the bed linens in hot water

 

 

69. A client is receiving oxygen therapy via a nasal cannula. When providing nursing care, which of the following interventions would be appropriate? A) Determine that adequate mist is supplied

B)  Inspect the nares and ears for skin breakdown

C)  Lubricate the tips of the cannula before insertion

D)  Maintain sterile technique when handling cannula

The correct answer is B: Inspect the nares and ears for skin breakdown

 

 

70. The nurse is caring for a client with Parkinson’s disease. The client spends over 1 hour to dress for scheduled therapies. What is the most appropriate action for the nurse to take in this situation?

A) Ask family members to dress the client

B)  Encourage the client to dress more quickly C) Allow the client the time needed to dress

D) Demonstrate methods on how to dress more quickly

The correct answer is C: Allow the client the time needed to dress

 

71. The nurse is assessing a 12 year-old who has Hemophilia A. Which finding would the nurse anticipate?

A) An excess of red blood cells

B)  An excess of white blood cells

C)  A deficiency of clotting factor VIII

D) A deficiency of clotting factors VIII and IX

The correct answer is C: A deficiency of clotting factor VIII

 

 

72. The nurse is assessing a newborn infant and observes low set ears, short palpebral fissures, flat nasal bridge and indistinct philtrum. A priority maternal assessment by the nurse should be to ask about

A) Alcohol use during pregnancy

B)  Usual nutritional intake

C)  Family genetic disorders D) Maternal and paternal ages

The correct answer is A: Alcohol use during pregnancy

 

 

73. A 2 month-old infant has both a cleft lip and palate which will be repaired in stages. In the immediate postoperative period for a cleft lip repair, which nursing approach should be the priority?

A)  Remove protective arm devices one at a time for short periods with supervision

B)  Initiate by mouth feedings when alert, with the return of the gag reflex

C)  Introduce to the parents how to cleanse the suture line with the prescribed protocol D) Position the infant on the back after feedings throughout the day

The correct answer is A: Remove protective arm devices one at a time for short periods with supervision

 

 

74. The new graduate nurse interviews for a position in a nursing department of a large health care agency, described by the interviewer as having shared governance. Which of these statements best illustrates the shared governance model? A) An appointed board oversees any administrative decisions

B)  Nursing departments share responsibility for client outcomes

C)  Staff groups are appointed to discuss nursing practice and client education issues D) Non-nurse managers supervise nursing staff in groups of units

The correct answer is B: Nursing departments share responsibility for client outcomes

 

 

75. The nurse is teaching childbirth preparation classes. One woman asks about her rights to develop a birthing plan. Which response made by the nurse would be best?

A)  “What is your reason for wanting such a plan?”

B)  “Have you talked with your health care provider about this?”

C)  “Let us discuss your rights as a couple.”

D)  “Write your ideal plan for the next class.”

The correct answer is C: “Let us discuss your rights as a couple.”

 

 

76. A client is admitted with the diagnosis of myocardial infarction (MI). Which of the

following lab values would be consistent with this diagnosis

A)  Low serum albumin

B)  High serum cholesterol

C)  Abnormally low white blood cell count

D)  Elevated creatinine phosphokinase (CPK )

The correct answer is D: Elevated CPK (creatinine phosphokinase)

 

 

77. A client tells the nurse he is fearful of planned surgery because of evil thoughts about a family member. What is the best initial response by the nurse?

A)  Call a chaplain

B)  Deny the feelings C) Cite recovery statistics

D) Listen to the client

The correct answer is D: Listen to the client

 

78. A 14 month-old had cleft palate surgical repair several days ago. The parents ask the nurse about feedings after discharge. Which lunch is the best example of an appropriate meal?

A)  Hot dog, carrot sticks, gelatin, milk

B)  Soup, blenderized soft foods, ice cream, milk

C)  Peanut butter and jelly sandwich, chips, pudding, milk

D)  Baked chicken, applesauce, cookie, milk

The correct answer is B: Soup, blenderized soft foods, ice cream, milk

 

 

79. The RN is planning care at a team meeting for a 2 month-old child in bilateral leg

casts for congenital clubfoot. Which of these suggestions by the PN should be considered the priority nursing goal following cast application? A) Infant will experience minimal pain

B) Muscle spasms will be relieved C) Mobility will be managed as tolerated D) Tissue perfusion will be maintained

The correct answer is D: Tissue perfusion will be maintained

 

 

80. The nurse would expect which eating disorder to have the greatest fluctuations in potassium?

A)  Binge eating disorder

B)  Anorexia nervosa

C)  Bulemia

D)  Purge syndrome

The correct answer is C: Bulemia

 

 

81. When planning the care for a young adult client diagnosed with anorexia nervosa which of these concerns should the nurse determine to be the priority for long term mobility?

A)  Digestive problems

B)  Amenorrhea

C)  Electrolyte imbalance

D)  Blood disorders

The correct answer is B: Amenorrhea

 

 

82. The nurse is planning care for a client with increased intracranial pressure. The best position for this client is

A)  Trendelenberg

B)  Prone

C)  Semi-Fowlers

D)  Side-lying with head flat

The correct answer is C: Semi-Fowlers

 

 

83. While performing an initial assessment on a newborn following a breech delivery, the nurse suspects hip dislocation. Which of the following is most suggestive of the abnormality?

A)  Flexion of lower extremities

B)  Negative Ortlani response

C)  Lengthened leg of affected side

D)  Irregular hip symmetry

The correct answer is D: Irregular hip symmetry

 

 

84. The nurse is caring for a client admitted to the hospital with right lower lobe (RLL) pneumonia. On assessment, the nurse notes crackles over the RLL. The client has significant pleuritic pain and is unable to take in a deep breath in order to cough effectively. Which nursing diagnosis would be most appropriate for this client based on this assessment data?

A)  Impaired gas exchange related to acute infection and sputum production

B)  Ineffective airway clearance related to sputum production and ineffective cough

C)  Ineffective breathing pattern related to acute infection D) Anxiety related to hospitalization and role conflict

The correct answer is B: Ineffective airway clearance related to sputum production and ineffective cough

 

 

85. A young child is admitted for treatment of lead poisoning. The nurse recognizes that the most serious effect of chronic lead poisoning is

A)  Central nervous system damage

B)  Moderate anemia

C)  Renal tubule damage

D)  Growth impairment

The correct answer is A: Central nervous system damage

 

 

86. At a nursing staff meeting, there is discussion of perceived inequities in weekend staff

assignments. As a follow-up, the nurse manager should initially A) Allow the staff to change assignments

B)  Clarify reasons for current assignments

C)  Help staff see the complexity of issues

D)  Facilitate creative thinking on staffing

The correct answer is D: Facilitate creative thinking on staffing

 

 

87. A client is admitted with a diagnosis of myocardial infarction (MI). The client is complaining of chest pain. The nurse knows that pain related to an MI is due to A)Insufficient oxygenation of the cardiac muscle

B)  Potential circulatory overload

C)  Left ventricular overload

D)  Electrolyte imbalance

The correct answer is A: Insufficient oxygenation of the cardiac muscle

 

 

88. A client was re-admitted to the hospital following a recent skull fracture. Which finding requires the nurse’s immediate attention?

A) Lethargy B) Agitation

C)  Ataxia

D)  Hearing loss

The correct answer is A: Lethargy

 

 

89. You are teaching a client about the patient controlled analgesia (PCA) planned for post-operative care. Which indicates further teaching may be needed by the client?

A)  “I will be receiving continuous doses of medication.”

B)  “I should call the nurse before I take additional doses.”

C)  “I will call for assistance if my pain is not relieved.”

D)  “The machine will prevent an overdose.”

The correct answer is B: “I should call the nurse before I take additional doses.”

 

 

90. When caring for a client with advanced cirrhosis of the liver, which nursing diagnosis should take priority?

A)  Risk for injury: hemorrhage

B)  Risk for injury related to peripheral neuropathy

C)  Altered nutrition: less than body requirements

D)  Fluid volume excess: ascites

The correct answer is A: Risk for injury: hemorrhage

 

 

91. The nurse is caring for a client with left ventricular heart failure. Which one of the following assessments is an early indication of inadequate oxygen transport?

A)  Crackles in the lungs

B)  Confusion and restlessness

C)  Distended neck veins

D)  Use of accessory muscles

The correct answer is B: Confusion and restlessness

 

 

92. On initial examination of a 15 month-old child with suspected otitis media, which group of findings would the RN anticipate finding?

A)  Periorbital edema, absent light reflex and translucent tympanic membrane

B)  Irritability, rhinorrhea, and bulging tympanic membrane

C)  Diarrhea, retracted tympanic membrane and enlarged parotid gland

D)  Vomiting, pulling at ears and pearly white tympanic membrane

The correct answer is B: Irritability, rinorrhea, and bulging tympanic membrane

 

 

93. A child with Tetralogy of Fallot visits the clinic several weeks before planned surgery. The nurse should give priority attention to

A)  Assessment of oxygenation

B)  Observation for developmental delays

C)  Prevention of infection

D)  Maintenance of adequate nutrition

The correct answer is A: Assessment of oxygenation

 

 

94. When teaching new parents to prevent Sudden Infant Death Syndrome (SIDS) what is the most important practice the nurse should instruct them to do? A) Place the infant in a supine or side lying position for sleep

B)  Do not allow anyone to smoke in the home

C)  Follow recommended immunization schedule

D)  Be sure to check infant every one hour

The correct answer is A: Place the infant in a supine or side lying position for sleep

 

 

95. A client is admitted with a distended bladder due to the inability to void. The nurse obtains an order to catheterize the client knowing that gradual emptying is preferred over complete emptying because it

A)  Reduces the potential for renal collapse

B)  Reduces the potential for shock

C)  Reduces the intensity of bladder spasms

D)  Prevents bladder atrophy

The correct answer is B: Reduces the potential for shock

 

 

96. The nurse is assessing a client with a deep vein thrombosis. Which of the following signs and/or symptoms would the nurse anticipate finding?

A)  Rapid respirations

B)  Diaphoresis

C)  Swelling of lower extremity

D)  Positive Babinski’s sign

The correct answer is C: Swelling of lower extremity

 

 

97. A 6 year-old female is diagnosed with recurrent urinary tract infections (UTI). Which one of the following instructions would be best for the nurse to tell the caregiver?

A)  Increase bladder tone by delaying voiding

B)  When laundering clothing, rinse several times C) Use plain water for the bath, shampooing hair last

D) Have the child use antibacterial soaps while bathing

The correct answer is C: Use plain water for the bath, shampooing hair last

 

 

98. A woman comes to the antepartum clinic for a routine prenatal examination. She is 12 weeks pregnant with her second child. Which of the following shows proper documentation of the client’s obstetric history by the nurse?

A)  Para 2, Gravida 1

B)  Nulligravida 2, Para 1

C)  Primagravida 1, Para 1

D)  Gravida 2, Para 1

The correct answer is D: Gravida 2, Para 1

 

 

99. On admission to the hospital a client with an acute asthma episode has intermittent nonproductive coughing and a pulse oximeter reading of 88%. The client states, “I feel like this is going to be a bad time this admission. I wish I would not have gone into that bar with all

those people who smoke last night.” Which nursing diagnoses would be most important for this client?

A) Anxiety related to hospitalization

B)  Ineffective airway clearance related to potential thick secretions

C)  Altered health maintenance related to preventative behaviors associated with asthma D) Impaired gas exchange related to broncho constriction and mucosal edema

The correct answer is D: Impaired gas exchange related to broncho constriction and mucosal edema

 

 

100. A client returned from surgery for a perforated appendix with localized peritonitis. In view of this diagnosis, how would the nurse position the client?

A)  Prone

B)  Dorsal recumbent

C)  Semi-Fowler

D)  Supine

The correct answer is C: Semi-Fowler

 

 

101. While caring for a client with infective endocarditis, the nurse must be alert for signs of pulmonary embolism. Which of the following assessment findings suggests this complication?

A)  Positive Homan’s sign

B)  Fever and chills

C)  Dyspnea and cough

D)  Sensory impairment

The correct answer is C: Dyspnea and cough

 

 

102. While assessing an Rh positive newborn whose mother is Rh negative, the nurse recognizes the risk for hyperbilirubinemia. Which of the following should be reported immediately?

A)  Jaundice evident at 26 hours

B)  Hematocrit of 55%

C)  Serum bilirubin of 12mg

D)  Positive Coomb’s test

The correct answer is C: Serum bilirubin of 12mg

 

 

103. The school nurse is called to the playground for an episode of mouth trauma. The nurse finds that the front tooth of a 9 year-old child has been avulsed (“knocked out”). After recovering the tooth, the initial response should be to A) Rinse the tooth in water before placing it in the socket

B)  Place the tooth in a clean plastic bag for transport to the dentist

C)  Hold the tooth by the roots until reaching the emergency room

D) Ask the child to replace the tooth even if the bleeding continues

The correct answer is A: Rinse the tooth in water before placing it in the socket

 

 

104. The nurse is caring for a 4 year-old child with a greenstick fracture. In explaining this type of fracture to the parents, the best response by the nurse should be that A) A child’s bone is more flexible and can be bent 45 degrees before breaking

B)  Bones of children are more porous than adults and often have incomplete breaks

C)  Compression of porous bones produces a buckle or torus type break

D)  Bone fragments often remain attached by a periosteal hinge

The correct answer is B: Bones of children are more porous than adults and often have incomplete breaks

 

 

105. During the beginning shift assessment of a client with asthma and is receiving

oxygen per nasal cannula at 2 liters per minute, the nurse would be most concerned about which unreported finding?

A)  Pulse oximetry reading of 89%

B)  Crackles at the base of the lungs on auscultation

C)  Rapid shallow respirations with intermittent wheezes

D)  Excessive thirst with a dry cracked tongue

The correct answer is C: Rapid shallow respirations with intermittent wheezes

 

 

106. During the care of a client with Legionnaire’s disease, which finding would require the nurse’s immediate attention? A) Pleuritic pain on inspiration

B)  Dry mucus membranes in the mouth

C)  A decrease in respiratory rate from 34 to 24 D) Decrease in chest wall expansion

The correct answer is D: Decrease in chest wall expansion

 

 

107. A child and his family were exposed to Mycobacterium tuberculosis about 2 months ago, to confirm the presence or absence of an infection, it is most important for all family members to have a A) Chest x-ray

B)  Blood culture

C)  Sputum culture

D)  PPD intradermal test

The correct answer is D: PPD intradermal test

 

 

108. The nurse is assigned to a client with Parkinson’s disease. Which findings would the nurse anticipate?

A)  Non intention tremors and urgency with voiding

B)  Echolalia and a shuffling gait

C)  Muscle spasm and a bent over posture

D)  Intention tremor and jerky movement of the elbows

The correct answer is B: Echolalia and a shuffling gait

 

 

109. Which of these statements by the nurse is incorrect to use to reinforce information about cancers to a group of young adults?

A)     You can reduce your risk of this serious type of stomach cancer by eating lots of fruits and vegetables, limiting all meat, and avoiding nitrate-containing foods.

B)     Prostate cancer is the most common cancer in American men with results to threaten sexuality and life.

C)     Colorectal cancer is the second-leading cause of cancer-related deaths in the United States.

D)     Lung cancer is the leading cause of cancer deaths in the United States. Yet it’s the most preventable of all cancers.

The correct answer is A. It is recommended that only red meat limited for the prevention of stomach cancer

 

110. A 67 year-old client is admitted with substernal chest pain with radiation to the jaw. His admitting diagnosis is Acute Myocardial Infraction (MI). The priority nursing diagnosis for this client during

the immediate 24 hours is

A)  Constipation related to immobility

B)  High risk for infection

C)  Impaired gas exchange

D)  Fluid volume deficit

The correct answer is C: Impaired gas exchange

 

 

111. With an alert of an internal disaster and the need for beds, the charge nurse is asked to list clients who are potential discharges within the next hour. Which client should the charge nurse select?

A) An elderly client who has had type 2 diabetes for over 20 years, admitted with diabetic ketoacidosis 24 hours ago

B)  An adolescent admitted the prior night with Tylenol intoxication

C)  A middle aged client with an internal automatic defibrillator and complaints of

“passing out at unknown times” admitted yesterday

D) A school age child diagnosed with suspected bacterial meningitis and was admitted at the change of shifts

The correct answer is A: An elderly client who has had type 2 diabetes for over 20 years, admitted with diabetic ketoacidosis 24 hours ago

 

 

112. The nurse is assessing a newborn the day after birth. A high pitched cry, irritability and lack of interest in feeding are noted. The mother signed her own discharge against medical advice. What intervention is appropriate nursing care?

A)  Reduce the environmental stimuli

B)  Offer formula every 2 hours

C)  Talk to the newborn while feeding D) Rock the baby frequently

The correct answer is A: Reduce the environmental stimuli

113. A hospitalized child suddenly has a seizure while his family is visiting. The nurse notes whole body rigidity followed by general jerking movements. The child vomits immediately after the seizure. A priority nursing diagnosis for the child is A) High risk for infection related to vomiting

B) Altered family processes related to chronic illness C) Fluid volume deficit related to vomiting

D) Risk for aspiration related to loss of consciousness

The correct answer is D: Risk for aspiration related to loss of consciousness

 

 

114. A 4 month-old child taking digoxin (Lanoxin) has a blood pressure of 92/78; resting pulse of 78; respirations 28 and a potassium level of 4.8 mEq/L. The client is irritable and has vomited twice since the morning dose of digoxin. Which finding is most indicative of digoxin toxicity? * A) Bradycardia

B)  Lethargy

C)  Irritability

D)  Vomiting

The correct answer is A: Bradycardia

 

 

115. A Hispanic client confides in the nurse that she is concerned that staff may give her newborn the “evil eye.” The nurse should communicate to other personnel that the appropriate approach is to

A)  Touch the baby after looking at him

B)  Talk very slowly while speaking to him

C)  Avoid touching the child

D)  Look only at the parents

The correct answer is A: Touch the baby after looking at him

 

 

116. A client is admitted for COPD. Which finding would require the nurse’s immediate attention?

A)  Nausea and vomiting

B)  Restlessness and confusion C) Low-grade fever and cough

D) Irritating cough and liquefied sputum

The correct answer is B: Restlessness and confusion

 

 

117. A young adult male has been diagnosed with testicular cancer. Which of these statements by this client would need to be explored by the nurse to clarify information?

A)    This surgical procedure involves removing one or both testicles through a cut in the groin. My lymph nodes in my lower belly also may be removed.

B)     I have a good chance to regain my fertility later. However if I am concerned, I can have my sperm frozen and preserved (cryopreserved) before chemotherapy. C) If I have cancer at stage 3 it means I have less involvement of the cancer.

D) After the surgical removal of a testicle, I can have an artificial testicle (prosthesis) placed inside my scrotum. This artificial implant has the weight and feel of a normal testicle.

The correct answer is C: If I have cancer at stage 3 it means I have less involvement of the cancer.

 

 

118. A newly appointed nurse manager is having difficulties with time management. Which advice from an experienced manager should the new manager do initially? A) Set daily goals and establish priorities for each hour and each day.

B)  Ask for additional assistance when you feel overwhelmed.

C)  Keep a time log of your day in hourly blocks for at least 1 week.

D)  Complete each task before beginning another activity in selected instances.

The correct answer is C: Keep a time log of your day in hourly blocks for at least 1 week.

 

 

119. The nurse and a student nurse are discussing the specific points about infants born to HBsAg-positive mothers. Which of these comments by the student indicates a need for clarification of

information?

A) “The infant will get the hepititis B vaccine (HepB) and the hepatitis B immune globulin within 12 hours at birth at separate injection sites.”

B)  “The second dose can be given at 1 to 2 months of age.”

C)  “The third dose should be given at least 16 weeks from the second dose.” D) “The last dose in the series is not to be given before age 24 weeks.”

The correct answer is C: “The third dose should be given at least 16 weeks from the second dose.”

 

 

120. A 74 year-old male is admitted due to inability to void. He has a history of an enlarged prostate and has not voided in 14 hours. When assessing for bladder distention, the best method for the nurse to use is to assess for

A)  Rebound tenderness

B)  Left lower quadrant dullness

C)  Rounded swelling above the pubis

D)  Urinary discharge

The correct answer is C: Rounded swelling above the pubis

 

  

121. Which one of the following statements, if made by the client, indicates teaching about Inderal (propranolol) has been effective?

A)  “I may experience seizures if I stop the medication apruptly.”

B)  " I may experience an increase in my heart rate for a few weeks."

C)  ” I can expect to feel nervousness the first few weeks."

D)  “ I can have a heart attack if I stop this medication suddenly."

The correct answer is D: “ I can have a heart attack if I stop this medication suddenly."

 

 

122. A 6 month-old infant who is being treated for developmental dysplasia of the hip has

been placed in a hip spica cast. The nurse should teach the parents to A) Gently rub the skin with a cotton swab to relieve itching B) Place the favorite books and push-pull toys in the crib

C) To check every few hours for the next day or 2 for swelling in the baby’s feet D) Turn the baby with the abduction stabilizer bar every 2 hours

The correct answer is C: To check frequently for swelling in the baby'’s feet

 

 

123. The nurse is teaching a client with cardiac disease about the anatomy and physiology of the heart. Which is the correct pathway of blood flow through the heart?

A)  Right ventricle, left ventricle, right atrium, left atrium

B)  Left ventricle, right ventricle, left atrium, right atrium

C)  Right atrium, right ventricle, left atrium, left ventricle

D)  Right atrium, left atrium, right ventricle, left ventricle

The correct answer is C: Right atrium, right ventricle, left atrium, left ventricle

 

 

124. The nurse manager has a nurse employee who is suspected of a problem with chemical dependency. Which intervention would be the best approach by the nurse manager?

A)  Confront the nurse about the suspicions in a private meeting

B)  Schedule a staff conference, without the nurse present, to collect information

C)  Consult the human resources department about the issue and needed actions

D)  Counsel the employee to resign to avoid investigation

The correct answer is C: Consult the human resources department about the issue and needed actions

 

 

125. The nurse would teach a client with Raynaud’s phenomenon that it is most important to

A)  Stop smoking

B)  Keep feet dry

C)  Reduce stress

D)  Avoid caffeine

The correct answer is A: Stop smoking

 

 

126. The nurse is caring for a client with status epileptics. The most important nursing assessment of this client is A) Intravenous drip rate

B)  Level of consciousness

C)  Pulse and respiration

D)  Injuries to the extremities

The correct answer is B: Level of consciousness

 

 

127. A client has been admitted for meningitis. In reviewing the laboratory analysis of

cerebrospinal fluid (CSF), the nurse would expect to note A) High protein

B)  Clear color

C)  Elevated sed rate

D)  Increased glucose

The correct answer is A: High protein

 

 

128. The hospital is planning to downsize and eliminate a number of staff positions as a cost-saving measure. To assist staff in this change process, the nurse manager is preparing for the “unfreezing” phase of change. With this approach and phase the nurse manager should

A)  Discuss with the staff how to deal with any defensive behavior

B)  Explain to the unit staff why change is necessary

C)  Assist the staff during the acceptance of the new changes

D)  Clarify what the changes mean to the community and hospital

The correct answer is B: Explain to the unit staff why the change is necessary

 

 

129. Which of these tests with frequency would the nurse expect to monitor for the evaluation of clients with poor glycemic control in persons aged 18 and older?

A)  A glycosylated hemoglobin (A1c) should be performed during an initial assessment and during follow-up assessments, which should occur at no longer than 3-month intervals

B)  A glycosylated hemoglobin is to be obtained at least twice a year

C)  A fasting glucose and a glycosylated hemoglobin is to be obtained at 3 months intervals after the initial assessment

D)  A glucose tolerance test, a fasting glucose and a glycosylated hemoglobin should be obtained at 6-month intervals after the initial assessment

The correct answer is A: A glycosylated hemoglobin (A1c) should be performed during an initial assessment and during follow-up assessments, which should occur at no longer than three-month intervals

 

130. At a routine health assessment, a client tells the nurse that she is planning a pregnancy in the near future. She asks about preconception diet changes. Which of the statements made by the nurse is best? A) “Include fibers in your daily diet.”

B) “Increase green leafy vegetable intake.” C) “Drink a glass of milk with each meal.”

D) “Eat at least 1 serving of fish weekly.”

The correct answer is B: “Increase green leafy vegetable intake.”

.

 

131. A client comes into the community health center upset and crying stating “I will die of cancer now that I have this disease.” And then the client hands the nurse a paper with one word written on it:

“Pheochromocytoma.” Which response should the nurse state initially?

A)    Pheochromocytomas usually aren’t cancerous (malignant). But they may be associated with cancerous tumors in other endocrine glands such as the thyroid (medullary carcinoma of the thyroid).

B)     This problem is diagnosed by blood and urine tests that reveal elevated levels of adrenaline and noradrenaline.

C)     Computerized tomography (CT) or magnetic resonance imaging (MRI) are used to detect an adrenal tumor.

D)    You probably have had episodes of sweating, heart pounding and headaches. The correct answer is A: Pheochromocytomas usually aren'’t cancerous (malignant). But they may be associated with cancerous tumors in other endocrine glands such as the thyroid (medullary carcinoma of the thyroid).

 

 

132. A client with chronic congestive heart failure should be instructed to contact the home health nurse if which finding occurs?

A) Weight gain of 2 pounds or more in a 48 hour period B) Urinating 4 to 5 times each day

C)  A significant decrease in appetite

D) Appearance of non-pitting ankle edema

The correct answer is A: Weight gain of 2 pounds or more in a 48 hour period

.

 

133. The nurse is caring for a client on mechanical ventilation. When performing

endotracheal suctioning, the nurse will avoid hypoxia by

A)  Inserting a fenestrated catheter with a whistle tip without suction

B)  Completing suction pass in 30 seconds with pressure of 150 mm Hg

C)  Hyper oxygenating with 100% O2 for 1 to 2 minutes before and after each suction pass

D)  Minimizing suction pass to 60 seconds while slowly rotating the lubricated catheter The correct answer is C: Hyper oxygenating with 100% O2 for 1-2 minutes before and after each suction pass

 

 

134. A female client diagnosed with genital herpes simplex virus 2 (HSV 2) complains of dysuria, dyspareunia, leukorrhea and lesions on the labia and perianal skin. A primary nursing action with the focus of comfort should be to

A)  Suggest 3 to 4 warm sitz baths per day

B)  Cleanse the genitalia twice a day with soap and water

C)  Spray warm water over genitalia after urination

D)  Apply heat or cold to lesions as desired

The correct answer is A: Encourage 3 to 4 warm sitz baths per day

 

 

135. Which finding would be the most characteristic of an acute episode of reactive airway disease?

A)  Auditory gurgling

B)  Inspiratory laryngeal stridor

C)  Auditory expiratory wheezing

D)  Frequent dry coughing

The correct answer is C: Wheezing on expiration

 

 

136. Which tasks, if delegated by the new charge nurse to a unlicensed assistive personnel (UAP), would require intervention by the nurse manager? A) To help an elderly client to the bathroom.

B)  To empty a foley catheter bag.

C)  To bathe a woman with internal radon seeds.

D)  To feed a 2 year-old with a broken arm.

The correct answer is C: To bathe a woman with internal radon seeds.

 

 

137. An 82 year-old client is prescribed eye drops for treatment of glaucoma. What assessment is needed before the nurse begins teaching proper administration of the medication?

A)  Determine third party payment plan for this treatment

B)  The client’s manual dexterity

C)  Proximity to health care services

D)  Ability to use visual assistive devices

The correct answer is B: The client’s manual dexterity

 

 

138. The nurse uses the DRG (Diagnosis Related Group) manual to

A)  Classify nursing diagnoses from the client’s health history

B)  Identify findings related to a medical diagnosis

C)  Determine reimbursement for a medical diagnosis

D)  Implement nursing care based on case management protocol

The correct answer is C: Determine reimbursement for a medical diagnosis

 

 

139. The community health nurse has been following the care for an adolescent with a history of morbid obesity, asthma, hypertension and is 22 weeks in to a pregnancy. Which of these lab reports sent to the clinic need to be called to the teens health care provider within the next hour?

A)  Hemoblobin 11 g/L and calcium 6 mg/dl

B)  Magnesium 0.8 mEq/L and creatinine 3 mg/dl

C)  Blood urea nitrogen 28 and glucose 225 mg/dl

D)  Hematocrit 33% and platelets 200,000

The correct answer is B: Magnesium 0.8 mEq/L and creatinine 3 mg/dl

 

 

140. The nurse has identified what appears to be ventricular tachycardia on the cardiac monitor of a client being evaluated for possible myocardial infarction. The first action the nurse would perform is to

A)  Begin cardiopulmonary resuscitation

B)  Prepare for immediate defibrillation

C)  Notify the “Code” team and health care provider

D)  Assess airway breathing and circulation

The correct answer is D: Assess airway breathing and circulation

 

 

141. To prevent keratitis in an unconscious client, the nurse should apply moisturizing ointment to the

A)  Finger and toenail quicks

B)  Eyes

C)  Perianal area

D)  External ear canals

The correct answer is B: Eyes

 

 

142. The nurse is caring for a child with cystic fibrosis. The nurse would anticipate that the child would be deficient in which vitamins?

A)  B, D, and K

B)  A, D, and K

C)  A, C, and D

D)  A, B, and C

The correct answer is B: A, D, and K

 

 

143. The nurse is teaching a 27 year-old client with asthma about management of their therapeutic regime. Which statement would indicate the need for additional instruction? A) “I should monitor my peak flow every day.”

B)  “I should contact the clinic if I am using my medication more often.”

C)  “I need to limit my exercise, especially activities such as walking and running.” D) “I should learn stress reduction and relaxation techniques.”

The correct answer is C: “I need to limit my exercise, especially activities such as walking and running.”

 

 

144. While caring for a child with Reye’s Syndrome, the nurse should give which action the highest priority? A) Monitor intake and output

B)  Provide good skin care

C)  Assess level of consciousness

D) Assist with range of motion

The correct answer is C: Assess level of consciousness

 

 

145. A newborn presents with a pronounced cephalic hematoma following a birth in the posterior position. Which nursing diagnosis should guide the plan of care?

A)  Pain related to periosteal injury

B)  Impaired mobility related to bleeding

C)  Parental anxiety related to knowledge deficit D) Injury related to inter cranial hemorrhage

The correct answer is C: Parental anxiety related to knowledge deficit

146. A confused client has been placed in physical restraints by order of the health care provider. Which task could be assigned to an unlicensed assistive personnel (UAP)?

A)  Assist the client with activities of daily living

B)  Monitor the clients physical safety

C)  Evaluate for basic comfort needs

D)  Document mental status and muscle strength

The correct answer is A: Assist with activities of daily living

 

 

147. A client is scheduled to have a blood test for cholesterol and triglycerides the next day. The nurse would tell the client

A)  “Be sure and eat a fat-free diet until the test.”

B)  “Do not eat or drink anything but water for 12 hours before the blood test.”

C)  “Have the blood drawn within 2 hours of eating breakfast.”

D)  “Stay at the laboratory so 2 blood samples can be drawn an hour apart.”

The correct answer is B: “Do not eat or drink anything but water for 12 hours before the blood test.”

 

 

148. A client who is terminally ill has been receiving high doses of an opiod analgesic for the past month. As death approaches and the client becomes unresponsive to verbal stimuli,what orders would the nurse expect from the health care provider? A) Decrease the analgesic dosage by half

B)  Discontinue the analgesic

C)  Continue the same analgesic dosage

D)  Prescribe a less potent drug

The correct answer is C: Continue the same analgesic dosage

 

 

149. Which of these clients would the triage nurse request for the health care provider to examine immediately?

A) A 5 month-old infant who has audible wheezing and grunting

B)  An adolescent who has soot over the face and shirt

C)  A middle-aged man with second degree burns over the right hand

D) A toddler with singed ends of long hair that extends to the waist

The correct answer is A: A 5 month-old infant who has audible wheezing and grunting

 

 

150. An infant has just returned from surgery for placement of a gastrostomy tube as an initial treatment for trachea esophageal fistula. The mother asks:”When can the tube can be used for feeding?” The nurse’s best response would be which of these comments? A) Feedings can begin in 5 to 7 days.

B)  The use of the feeding tube can begin immediately.

C)  The stomach contents and air must be drained first.

D)  The incision healing must be complete before feeding.

The correct answer is C: Stomach contents and air must be drained first

 

 

151. A pre-term baby develops nasal flaring, cyanosis and diminished breath sounds on one side. The provider’s diagnosis is spontaneous pneumothorax. Which procedure should the nurse prepare for first? A) Cardiopulmonary resuscitation

B)  Insertion of a chest tube

C)  Oxygen therapy

D) Assisted ventilation

The correct answer is B: Insertion of a chest tube

 

 

152. The nurse is caring for a 75 year old client in congestive heart failure. Which finding suggests that digitalis levels should be reviewed?

A)  Extreme fatigue

B)  Increased appetite

C)  Intense itching

D)  Constipation

The correct answer is A: Extreme fatigue

 

 

153. The nurse is teaching a client with atrial fibrillation about the use of Coumadin (warfarin) at home. Which of these should be emphasized to the client to avoid?

A)  Large indoor gatherings

B)  Exposure to sunlight

C)  Active physical exercise

D)  Foods rich in vitamin K

The correct answer is D: Foods rich in vitamin K

 

 

154. A nurse caring for premature newborns in an intensive care setting carefully monitors oxygen concentration. What is the most common complication of this therapy? A) Intraventricular hemorrhage

B)  Retinopathy of prematurity

C)  Bronchial pulmonary dysplasia

D)  Necrotizing enterocolitis

The correct answer is B: Retinopathy of prematurity

 

 

155. A nurse manager is using the technique of brainstorming to help solve a problem. One nurse criticizes another nurse’s contribution and begins to find objections to the suggestion. The nurse manager’s best response is to

A)  Let’s move on to a new action that deals with the problem.

B)  I think you need to reserve judgment until after all suggestions are offered.

C)  Very well thought out. Your analytic skills and interest are incredible.

D)  Let’s move to the ‘what if…’ as related to these objections for an exploration of spin off ideas.

The correct answer is D: Let’s move to the ‘what if…’ as related to these objections for an exploration of spin off ideas.

 

 

156. The nurse is caring for an acutely ill 10 year-old client. Which of the following assessments would require the nurses immediate attention?

A)  Rapid bounding pulse

B)  Temperature of 38.5 degrees Celsius

C)  Profuse Diaphoresis

D)  Slow, irregular respirations

The correct answer is D: Slow, irregular respirations

 

 

157. A child is diagnosed with poison ivy. The mother tells the nurse that she does not know how her child contracted the rash since he had not been playing in wooded areas. As the nurse asks questions about possible contact, which of the following would the nurse recognize as highest risk for exposure?

A)  Playing with toys in a back yard flower garden

B)  Eating small amounts of grass while playing “farm”

C)  Playing with cars on the pavement near burning leaves

D)  Throwing a ball to a neighborhood child who has poison ivy

The correct answer is C: Playing with cars on the pavement near burning leaves

 

 

158. The nurse is teaching a group of adults about modifiable cardiac risk factors. Which of the following should the nurse focus on first?

A)  Weight reduction

B)  Stress management

C)  Physical exercise

D)  Smoking cessation

The correct answer is D: Smoking cessation

 

159. The nurse is caring for a 5 year-old child who has the left leg in skeletal traction. Which of the following activities would be an appropriate diversional activity? A) Kicking balloons with right leg

B)  Playing “Simon Says”

C)  Playing hand held games

D)  Throw bean bags

The correct answer is C: Playing hand held games

 

 

160. The nurse is assessing a client with portal hypertension. Which of the following findings would the nurse expect?

A)  Expiratory wheezes

B)  Blurred vision

C)  Acites

D)  Dilated pupils

The correct answer is C: Acites

  

2018 HESI EXIT V6

 

1. A parent tells the nurse that their 6 year-old child who normally enjoys school, has not been doing well since the grandmother died 2 months ago. Which statement most accurately describes thoughts on

death and dying at this age?

A)  Death is personified as the bogeyman or devil

B)  Death is perceived as being irreversible

C)  The child feels guilty for the grandmother’s death D) The child is worried that he, too, might die

The correct answer is A: Death is personified as the bogeyman or devil

 

2. A 67 year-old client with non-insulin dependent diabetes should be instructed to contact the out-patient clinic immediately if the following findings are present

A) Temperature of 37.5 degrees Celsius with painful urination B) An open wound on their heel

C)  Insomnia and daytime fatigue

D)  Nausea with 2 episodes of vomiting

The correct answer is B: An open wound on their heel

 

 

3. The nurse admits an elderly Mexican-American migrant worker after an accident that

occurred during work. To facilitate communication the nurse should initially

A)  Request a Spanish interpreter

B)  Speak through the family or co-workers

C)  Use pictures, letter boards, or monitoring D) Assess the client’s ability to speak English The correct answer is D: Assess the client'’s ability to speak English

 

 

4. In assessing a post partum client, the nurse palpates a firm fundus and observes a constant trickle of bright red blood from the vagina. What is the most likely cause of these findings? A) Uterine atony

B)  Genital lacerations

C)  Retained placenta

D)  Clotting disorder

The correct answer is B: Genital lacerations

 

 

5. The nurse notes an abrupt onset of confusion in an elderly patient. Which of the following recently-ordered medications would most likely contribute to this change?

A)  Anticoagulant

B)  Liquid antacid

C)  Antihistamine

D)  Cardiac glycoside

The correct answer is C: Antihistamine

 

 

6. The nurse is caring for a client with active tuberculosis who has a history of noncompliance. Which of the following actions by the nurse would represent appropriate care for this client?

A)  Instruct the client to wear a high efficiency particulate air mask in public places.

B)  Ask a family member to supervise daily compliance

C)  Schedule weekly clinic visits for the client

D)  Ask the health care provider to change the regimen to fewer medications

The correct answer is B: Ask a family member to supervise daily compliance

 

7. The nurse manager identifies that time spent by staff in charting is excessive, requiring overtime for completion. The nurse manager states that “staff will form a task force to investigate and develop potential solutions to the problem, and report on this at the next

staff meeting.” The nurse manager’s leadership style is best described as A) Laissez-faire

B)  Autocratic

C)  Participative

D)  Group

The correct answer is C: Participative

 

8. A nursing student asks the nurse manager to explain the forces that drive health care

reform. The appropriate response by the nurse manager should include A) The escalation of fees with a decreased reimbursement percentage

B)  High costs of diagnostic and end-of-life treatment procedures

C)  Increased numbers of elderly and of the chronically ill of all ages

D)  A steep rise in health care provider fees and in insurance premiums The correct answer is A: The escalation of fees with a decreased reimbursement percentage

 

 

9. A client with hepatitis A (HAV) is newly admitted to the unit. Which action would be the priority to include in the plan of care within the initial 24 hours for this client?

A)  Wear masks with shields if potential splash

B)  Use disposable utensils and plates for meals

C)  Wear gown and gloves during client contact

D)  Provide soft easily digested food with frequent snacks

The correct answer is C: Wear gown and gloves during client contact

 

 

10. A client has been taking alprazolam (Xanax) for 3 days. Nursing assessment should reveal which expected effect of the drug? A) Tranquilization, numbing of emotions

B)  Sedation, analgesia

C)  Relief of insomnia and phobias

D)  Diminished tachycardia and tremors associated with anxiety

The correct answer is A: Tranquilization, numbing of emotions

 

 

11. The nurse observes a staff member caring for a client with a left unilateral mastectomy. The nurse would intervene if she notices the staff member is A) Advising client to restrict sodium intake

B)  Taking the blood pressure in the left arm

C)  Elevating her left arm above heart level

D)  Compressing the drainage device

The correct answer is B: Taking the blood pressure in the left arm

 

 

12. A 70 year-old post-operative client has elevated serum BUN, Hct, Cl, and Na+. Creatinine and K+ are within normal limits. The nurse should perform additional assessments to confirm that an actual

problem is:

A)  Impaired gas exchange

B)  Metabolic acidosis

C)  Renal insufficiency

D)  Fluid volume deficit

The correct answer is D: Fluid volume deficit

 

 

13. The nurse is providing foot care instructions to a client with arterial insufficiency. The

nurse would identify the need for additional teaching if the client stated A) “I can only wear cotton socks.”

B)  “I cannot go barefoot around my house.”

C)  “I will trim corns and calluses regularly.”

D)  “I should ask a family member to inspect my feet daily.”

The correct answer is C: “I will trim corns and calluses regularly.”

 

 

14. A woman who delivered 5 days ago and had been diagnosed with preeclampsia calls

the hospital triage nurse hotline to ask for advice. She states “ I have had the worst headache for the past 2 days. It

pounds and by the middle of the afternoon everything I look at looks wavy. Nothing I have taken helps.” What should the nurse do next?

A) Advise the client that the swings in her hormones may have that effect. However, suggest for her to call her health care provider within the next day.

B)  Advise the client to have someone bring her to the emergency room as soon as possible

C)  Ask the client to stay on the line, get the address and send an ambulance to the home

D) Ask what the client has taken? How often? Ask about other specific complaints. The correct answer is C: Ask the client to stay on the line, get the address and send an ambulance to the home

 

 

15. The primary teaching for a client following an extracorporeal shock-wave lithotripsy (ESWL) procedure is

A) Drink 3000 to 4000 cc of fluid each day for one month B) Limit fluid intake to 1000 cc each day for one month

C)  Increase intake of citrus fruits to three servings per day

D)  Restrict milk and dairy products for one month

The correct answer is A: Drink 3000 to 4000 cc of fluid each day for 1 month

 

 

16. A client on warfarin therapy following coronary artery stent placement calls the clinic to ask if he can take Alka-Seltzer for an upset stomach. What is the best response by the nurse?

A)  Avoid Alka-Seltzer because it contains aspirin

B)  Take Alka-Seltzer at a different time of day than the warfarin

C)  Select another antacid that does not inactivate warfarin

D)  Use on-half the recommended dose of Alka-Seltzer

The correct answer is A: Avoid Alka-Seltzer because it contains aspirin

 

17. The nurse is working with parents to plan home care for a 2 year-old with a heart problem. A priority nursing intervention would be to

A)  Encourage the parents to enroll in cardiopulmonary resuscitation class

B)  Assist the parents to plan quiet play activities at home

C)  Stress to the parents that they will need relief care givers

D)  Instruct the parents to avoid contact with persons with infection The correct answer is A: Encourage the parents to enroll in cardiopulmonary resuscitation class

18. The nurse is caring for a client with Rheumatoid Arthritis. Which nursing diagnosis should receive priority in the plan of care? A) Risk for injury

B)  Self care deficit

C)  Alteration in comfort D) Alteration in mobility

The correct answer is C: Alteration in comfort

 

19. An unlicensed assistive staff member asks the nurse manager to explain the beliefs of a Christian Scientist who refuses admission to the hospital after a motor vehicle accident. The best response of the nurse would be which of these statements?

A)  “Spiritual healing is emphasized and the mind contributes to the cure.”

B)  “The primary belief is that dietary practices result in health or illness.” C) “Fasting and prayer are initial actions to take in physical injury.”

D) “Meditation is intensive in the initial 48 hours and daily thereafter.”

The correct answer is A: “Spiritual healing is emphasized and the mind contributes to the cure.”

 

 

20. In order to be effective in administering cardiopulmonary resuscitation to a 5 year- old, the nurse must

A)  Assess the brachial pulses

B)  Breathe once every 5 compressions

C)  Use both hands to apply chest pressure

D)  Compress 80-90 times per minute

The correct answer is B: Breathe once every 5 compressions

 

 

21. The nurse is providing home care for a client with heart failure and pulmonary edema.

Which nursing diagnosis should have priority in planning care? A) Impaired skin integrity related to dependent edema

B)  Activity intolerance related to oxygen supply and demand imbalance

C)  Constipation related to immobility

D)  Risk for infection related to ineffective mobilization of secretions

The correct answer is B: Activity Intolerance related to oxygen supply and demand imbalance

.  

22. For which of the following mother-baby pairs should the nurse review the Coomb’s test in preparation for administering RhO (D) immune globulin within 72 hours of birth? A) Rh negative mother with Rh positive baby

B)  Rh negative mother with Rh negative baby

C)  Rh positive mother with Rh positive baby

D)  Rh positive mother with Rh negative baby

The correct answer is A: Rh negative mother with Rh positive baby

 

 

23. An 80 year-old nursing home resident has a temperature of 101.6 degrees Fahrenheit rectally. This is a sudden change in an otherwise healthy client. Which should the nurse assess first?

A)  Lung sounds

B)  Urine output

C)  Level of alertness

D)  Appetite

The correct answer is C: Level of alertness

 

?  

25. What is the major purpose of community health research?

A)  Describe the health conditions of populations

B)  Evaluate illness in the community

C)  Explain the health conditions of families

D)  Identify the health conditions of the environment

The correct answer is A: Describe the health conditions of populations

 

 

26. The recent increase in the reported cases of active tuberculosis (TB) in the United States is attributed to which factor?

A)  The increased homeless population in major cities

B)  The rise in reported cases of positive HIV infections

C)  The migration patterns of people from foreign countries D) The aging of the population located in group homes

The correct answer is B: The rise in reported cases of positive HIV infections

 

 

27. A 15 month-old child comes to the clinic for a follow-up visit after hospitalization for treatment of Kawasaki Disease. The nurse recognizes that which of the following scheduled immunizations will be

delayed?

A)  MMR

B)  Hib

C)  IPV

D)  DtaP

The correct answer is A: MMR

 

  

28. The nurse is assessing a pregnant client in her third trimester. The parents are informed that the ultrasound suggests that the baby is small for gestational age (SGA). An earlier ultrasound indicated normal growth. The nurse understands that this change is most likely due to what factor?

A)  Sexually transmitted infection

B)  Exposure to teratogens C) Maternal hypertension

D) Chromosomal abnormalities

The correct answer is C: Maternal hypertension

 

29. After the shift report in a labor and delivery unit which of these clients would the nurse check first?

A) A middle aged woman with asthma and diabetes mellitus Type 1 has a BP of 150/94

B)  A middle aged woman with a history of two prior vaginal term births is 2 cm dilated

C)  A young woman is a grand multipara has cervical dilation of 4 cm and 50% effaced D) An adolescent who is 18 weeks pregnant has a report of no fetal heart tones and coughing up frothy sputum

The correct answer is D: An adolescent who is 18 weeks pregnant has a report of no fetal heart tones and coughing up frothy sputum

 

 

30. The nurse is caring for an 87 year-old client with urinary retention. Which finding should be reported immediately? A) Fecal impaction

B)  Infrequent voiding

C)  Stress incontinence

D)  Burning with urination

The correct answer is A: Fecal impaction

 

 

31. The nasogastric tube of a post-op gastrectomy client has stopped draining greenish liquid. The nurse should

A) Irrigate it as ordered with distilled water B) Irrigate it as ordered with normal saline

C) Place the end of the tube in water to see if the water bubbles D) Withdraw the tube several inches and reposition it

The correct answer is B: Irrigate it as ordered with normal saline

 

 

32. The parents of a child who has recently been diagnosed with asthma ask the nurse to explain the condition to them. The best response is “Asthma causes…

A)  the airway to become narrow and obstructs airflow.”

B)  air to be trapped in the lungs because the airways are dilated."

C)  the nerves that control respiration to become hyperactive."

D)  a decrease in the stress hormones which prevents the airways from opening."

The correct answer is A: the airway to become narrow and obstructs airflow."

 

 

33. The nurse is assessing a child with suspected lead poisoning. Which of the following assessments is the nurse most likely to find? A) Complaints of numbness and tingling in feet

B)  Wheezing noted when lung sound auscultated

C)  Excessive perspiration

D)  Difficulty sleeping

The correct answer is A: Complaints of numbness and tingling in feet

 

 

34. The nurse is caring for a client with end-stage heart failure. The family members are distressed about the client’s impending death. What action should the nurse do first? A) Explain the stages of death and dying to the family

B)  Recommend an easy-to-read book on grief

C)  Assess the family’s patterns for dealing with death

D) Ask about their religious affiliations

The correct answer is C: Assess the family'’s patterns for dealing with death

 

 

35. The nurse is caring for a client with Meniere’s disease. When teaching the client about

the disease, the nurse should explain that the client should avoid foods high in

A)  Calcium

B)  Fiber

C)  Sodium

D)  Carbohydrate

The correct answer is C: Sodium

 

 

36. The nurse is teaching a mother who will breast feed for the first time. Which of the following is a priority?

A)  Show her films on the physiology of lactation

B)  Give the client several illustrated pamphlets

C)  Assist her to position the newborn at the breast

D)  Give her privacy for the initial feeding

The correct answer is C: Assist her to position the newborn at the breast

 

  

37. The nurse is taking a health history from parents of a child admitted with possible Reye’s Syndrome. Which recent illness would the nurse recognize as increasing the risk to develop Reye’s Syndrome?

A)  Rubeola

B)  Meningitis

C)  Varicella

D)  Hepatitis

The correct answer is C: Varicella

 

 

38. While giving care to a 2 year-old client, the nurse should remember that the toddler’s tendency to say “no” to almost everything is an indication of what psychosocial skill?

A)  Stubborn behavior

B)  Rejection of parents

C)  Frustration with adults

D)  Assertion of control

The correct answer is D: Assertion of control

 

39. A postpartum client admits to alcohol use throughout the pregnancy. Which of the following newborn assessments suggests to the nurse that the infant has fetal alcohol syndrome?

A)  Growth retardation is evident

B)  Multiple anomalies are identified

C)  Cranial facial abnormalities are noted

D)  Prune belly syndrome is suspected

The correct answer is C: Cranial facial abnormalities are noted

 

 

40. The nurse is attending a workshop about caring for persons infected with Hepatitis. Which statement is correct when referring to the incidence rate for Hepatitis?

A)    The number of persons in a population who develop Hepatitis B during a specific period of time

B)     The total number of persons in a population who have Hepatitis B at a particular time C) The percentage of deaths resulting from Hepatitis B during a specific time D) The occurrence of Hepatitis B in the population at a particular time

The correct answer is A: The number of persons in a population who develop Hepatitis B during a specific period of time

41. A 36 year-old female client has a hemoglobin level of 14 g/dl and a hematocrit of 42% following a D&C. Which of the following would the nurse expect to find when assessing this client?

A)  Capillary refill less than 3 seconds

B)  Pale mucous membranes

C)  Respirations 36 breaths per minute

D)  Complaints of fatigue when ambulating

The correct answer is A: Capillary refill less than 3 seconds

 

 

42. The nurse is caring for a client suspected to have Tuberculosis (TB). Which of the following diagnostic tests is essential for determining the presence of active TB? The nurse is caring for a client suspected to have Tuberculosis (TB). Which of the following diagnostic tests is essential for determining the presence of active TB? A) Tuberculin skin testing

B)  Sputum culture

C)  White blood cell count D) Chest x-ray

The correct answer is B: Sputum culture

 

 

43. The nurse has been teaching an apprehensive primipara who has difficulty in initial nursing of the newborn. What observation at the time of discharge suggests that initial breast feeding is effective?

A)  The mother feels calmer and talks to the baby while nursing

B)  The mother awakens the newborn to feed whenever it falls asleep

C)  The newborn falls asleep after 3 minutes at the breast

D)  The newborn refuses the supplemental bottle of glucose water

The correct answer is A: The mother feels calmer and talks to the baby while nursing

 

 

44. The mother of a burned child asks the nurse to clarify what is meant by a third degree burn. The best response by the nurse is A) “The top layer of the skin is destroyed.”

B)  “The skin layers are swollen and reddened.”

C)  “All layers of the skin were destroyed in the burn.”

D)  “Muscle, tissue and bone have been injured.”

The correct answer is C: “All layers of the skin were destroyed in the burn.”

 

 

45. The nurse is taking a health history from a Native American client. It is critical that the nurse must remember that eye contact with such clients is considered

A)  Expected

B)  Rude

C)  Professional

D)  Enjoyable

The correct answer is B: Rude

 

 

46. A nurse is instructing a class for new parents at a local community center. The nurse would stress that which activity is most hazardous for an 8 month-old child?

A)  Riding in a car

B)  Falling off a bed

C)  Electrical outlets

D)  Eating peanuts

The correct answer is D: Eating peanuts

 

 

47. When teaching parents about sickle cell disease, the nurse should tell them that their child’s anemia is caused by

A)  Reduced oxygen capacity of cells due to lack of iron

B)  An imbalance between red cell destruction and production

C)  Depression of red and white cells and platelets

D)  Inability of sickle shaped cells to regenerate

The correct answer is B: An imbalance between red cell destruction and production

 

 

48. The nurse is assessing a newborn delivered at home by an admitted heroin addict.

Which of the following would the nurse expect to observe? A) Hypertonic neuro reflex

B)  Immediate CNS depression

C)  Lethargy and sleepiness

D)  Jitteriness at 24-48 hours

The correct answer is D: Jitteriness at 24-48 hours

 

 

49. The nurse is caring for a client with congestive heart failure. Which finding requires the nurse’s immediate attention?

A)  Pulse oximetry of 85%

B)  Nocturia

C)  Crackles in lungs D) Diaphoresis

The correct answer is A: Pulse oximetry of 85%

 

 

50. The nurse is assessing a young child at a clinic visit for a mild respiratory infection. Koplik spots are noted on the oral mucous membranes. The nurse should then assess which area of the body? A) Inspect the skin

B)  Auscultate breath sounds

C)  Evaluate muscle strength

D)  Investigate elimination patterns

The correct answer is A: Inspect the skin

 

 

51. Which action is most likely to ensure the safety of the nurse while making a home visit?

A)  Observation during the visit of no evidence of weapons in the home

B)  Prior to the visit, review client’s record for any previous entries about violence

C)  Remain alert at all times and leave if cues suggest the home is not safe D) Carry a cell phone, pager and/or hand held alarm for emergencies

The correct answer is C: Staying alert at all times and leaving if cues suggest the home is not safe

 

 

52. An adolescent client is admitted in respiratory alkalosis following aspirin overdose.

The nurse recognizes that this imbalance was caused by

A)  Tachypnea

B)  Acidic byproducts

C)  Vomiting and dehydration D) Hyperpyrexia

The correct answer is A: Tachypnea

 

 

53. The nurse discovers that the parents of a 2 year-old child continue to use an apnea monitor each night. The parents state: “We are concerned about the possible occurrence of sudden infant death

syndrome (SIDS).” In order to take appropriate action, the nurse must understand that A) The child is within the age group most susceptible to SIDS

B)  The peak age for occurrence of SIDS is 8 to 12 months of age

C)  The apnea monitor is not effective on a child in this age group

D)  95% of SIDS cases occur before 6 months of age

The correct answer is D: 95% percent of all SIDS cases occur before 6 months

 

 

54. As a client is being discharged following resolution of a spontaneous pneumothorax,

he tells the nurse that he is now going to Hawaii for a vacation. The nurse would warn him to avoid A) Surfing

B)  Scuba diving

C)  Parasailing

D)  Swimming

Review Information: The correct answer is B: Scuba diving

 

 

55. The nurse is providing diet instruction to the parents of a child with cystic fibrosis. The nurse would emphasize that the diet should be A) High calorie, low fat, low sodium

B)  High protein, low fat, low carbohydrate

C)  High protein, high calorie, unrestricted fat

D)  High carbohydrate, low protein, moderate fat

The correct answer is C: High protein, high calorie, unrestricted fat

 

 

56. A client had arrived in the USA from a developing country 1 week prior. The client is to be admitted to the medical surgical unit with a diagnosis of AIDS with a history of unintended weight loss, drug abuse, night sweats, productive cough and a “feeling of being hot all the time.” The nurse should assign the client to share a room with a client with the diagnosis of

A)   Acute tuberculosis with a productive cough of discolored sputum for over three months

B)    Lupus and vesicles on one side of the middle trunk from the back to the abdomen

C)    Pseudomembranous colitis and C. difficile.

D)   Exacerbation of polyarthritis with severe pain

The correct answer is A: Acute tuberculosis with a productive cough of discolored sputum for over three months

 

 

57. A client’s admission urinalysis shows the specific gravity value of 1.039. Which of the following assessment data would the nurse expect to find when assessing this client?

A)  Moist mucous membranes

B)  Urinary frequency

C)  Poor skin turgor

D)  Increased blood pressure

The correct answer is C: Poor skin turgor

 

 

58. Parents are concerned that their 11 year-old child is a very picky eater. The nurse

suggests which of the following as the best initial approach? A) Consider a liquid supplement to increase calories

B)  Discuss consequences of an unbalanced diet with the child

C)  Provide fruit, vegetable and protein snacks

D)  Encourage the child to keep a daily log of foods eaten

The correct answer is B: Discuss consequences of an unbalanced diet with the child

 

 

59. At a community health fair the blood pressure of a 62 year-old client is 160/96. The client states “My blood pressure is usually much lower.” The nurse should tell the client to

A)  go get a blood pressure check within the next 48 to 72 hours

B)  check blood pressure again in 2 months

C)  see the health care provider immediately

D)  visit the health care provider within 1 week for a BP check

The correct answer is A: go get a blood pressure check within the next 48 to 72 hours

 

 

60. A client is admitted to the emergency room with renal calculi and is complaining of moderate to severe flank pain and nausea. The client’s temperature is 100.8 degrees

Fahrenheit. The priority nursing goal for this client is

A)  Maintain fluid and electrolyte balance

B)  Control nausea

C)  Manage pain

D)  Prevent urinary tract infection

The correct answer is C: Manage pain The immediate goal of therapy is to alleviate the client’s pain.

 

61. An RN who usually works in a spinal rehabilitation unit is floated to the emergency department. Which of these clients should the charge nurse assign to this RN?

A)    A middle-aged client who says “I took too many diet pills” and “my heart feels like it is racing out of my chest.”

B)     A young adult who says “I hear songs from heaven. I need money for beer. I quit drinking 2 days ago for my family. Why are my arms and legs jerking?”

* C)An adolescent who has been on pain medications for terminal cancer with an initial assessment finding of pinpoint pupils and a relaxed respiratory rate of 10

D) An elderly client who reports having taken a “large crack hit” 10 minutes prior to walking into the emergency room

The correct answer is c: An adolescent who has been on pain medications for terminal cancer with an initial assessment finding of pinpoint pupils and a relaxed respiratory rate of 10

 

 

62. While planning care for a toddler, the nurse teaches the parents about the expected developmental changes for this age. Which statement by the mother shows that she understands the child’s developmental needs? A) “I want to protect my child from any falls.”

B)  “I will set limits on exploring the house.”

C)  “I understand the need to use those new skills.”

D)  “I intend to keep control over our child.”

The correct answer is C: “I understand the need to use those new skills.”

 

63. A client who is pregnant comes to the clinic for a first visit. The nurse gathers data about her obstetric history, which includes 3 year old twins at home and a miscarriage 10 years ago at 12 weeks

gestation. How would the nurse accurately document this information?

A)  Gravida 4 para 2

B)  Gravida 2 para 1

C)  Gravida 3 para 1 D) Gravida 3 para 2

The correct answer is C: Gravida 3 para 1

 

 

64. A child who ingested 15 maximum strength acetaminophen tablets 45 minutes ago is seen in the emergency department. Which of these orders should the nurse do first?

A)  Gastric lavage PRN

B)  Acetylcysteine (mucomyst) for age per pharmacy

C)  Start an IV Dextrose 5% with 0.33% normal saline to keep vein open D) Activated charcoal per pharmacy

The correct answer is A: Gastric lavage PRN

 

65. The nurse is preparing to administer an enteral feeding to a client via a nasogastric feeding tube. The most important action of the nurse is A) Verify correct placement of the tube

B)  Check that the feeding solution matches the dietary order

C)  Aspirate abdominal contents to determine the amount of last feeding remaining in stomach

D)  Ensure that feeding solution is at room temperature

The correct answer is A: Verify correct placement of the tube

 

 

66. The nurse anticipates that for a family who practices Chinese medicine the priority goal would be to A) Achieve harmony

B)  Maintain a balance of energy

C)  Respect life

D)  Restore yin and yang

The correct answer is D: Restore yin and yang

 

 

67. Which complication of cardiac catheterization should the nurse monitor for in the initial 24 hours after the procedure?

A)  angina at rest

B)  thrombus formation

C)  dizziness

D)  falling blood pressure

The correct answer is B: thrombus formation

 

 

68. A nurse prepares to care for a 4 year-old newly admitted for rhabdomyosarcoma. The nurse should alert the staff to pay more attention to the function of which area of the body? A) The muscles

B)  The cerebellum

C)  The kidneys

D)  The leg bones

The correct answer is A: All striated muscles

 

 

69. A client comes to the clinic for treatment of recurrent pelvic inflammatory disease.

The nurse recognizes that this condition most frequently follows which type of infection? A) Trichomoniasis B) Chlamydia

C)  Staphylococcus

D)  Streptococcus

The correct answer is B: Chlamydia

 

 

70. During the evaluation of the quality of home care for a client with Alzheimer’s disease, the priority for the nurse is to reinforce which statement by a family member? A) At least 2 full meals a day is eaten.

B)  We go to a group discussion every week at our community center.

C)  We have safety bars installed in the bathroom and have 24 hour alarms on the doors.

D)  The medication is not a problem to have it taken 3 times a day.

The correct answer is C: We have safety bars installed in the bathroom and have 24 hour alarms on the doors.

 

 

71. The nurse is caring for a client with a venous stasis ulcer. Which nursing intervention would be most effective in promoting healing? A) Apply dressing using sterile technique

B)  Improve the client’s nutrition status

C)  Initiate limb compression therapy

D)  Begin proteolytic debridement

The correct answer is B: Improve the client'’s nutrition status

 

 

72. During an assessment of a client with cardiomyopathy, the nurse finds that the systolic blood pressure has decreased from 145 to 110 mm Hg and the heart rate has risen from 72 to 96 beats per minute and the client complains of periodic dizzy spells. The nurse instructs the client to

A)  Increase fluids that are high in protein

B)  Restrict fluids

C)  Force fluids and reassess blood pressure

D)  Limit fluids to non-caffeine beverages

The correct answer is C: Force fluids and reassess blood pressure

 

 

73. Which individual is at greatest risk for developing hypertension? A) 45 year-old African American attorney

B)  60 year-old Asian American shop owner

C)  40 year-old Caucasian nurse

D)  55 year-old Hispanic teacher

The correct answer is A: 45 year-old African American attorney

 

74. The nurse is caring for a client with a serum potassium level of 3.5 mEq/L. The client is placed on a cardiac monitor and receives 40 mEq KCL in 1000 ml of 5% dextrose in water IV. Which of the following EKG patterns indicates to the nurse that the infusions should be discontinued? A) Narrowed QRS complex

B)  Shortened “PR” interval

C)  Tall peaked T waves

D)  Prominent “U” waves

The correct answer is C: Tall peaked T waves

 

 

75. A client has been taking furosemide (Lasix) for the past week. The nurse recognizes which finding may indicate the client is experiencing a negative side effect from the medication?

A)  Weight gain of 5 pounds

B)  Edema of the ankles

C)  Gastric irritability

D)  Decreased appetite

The correct answer is D: Decreased appetite

 

 

76. Which of these statements best describes the characteristic of an effective reward- feedback system?

A)  Specific feedback is given as close to the event as possible

B)  Staff are given feedback in equal amounts over time

C)  Positive statements are to precede a negative statement

D)  Performance goals should be higher than what is attainable

The correct answer is A: Specific feedback is given as close to the event as possible

 

 

77. The nurse practicing in a maternity setting recognizes that the post mature fetus is at risk due to

A)  Excessive fetal weight

B)  Low blood sugar levels

C)  Depletion of subcutaneous fat

D)  Progressive placental insufficiency

The correct answer is D: Progressive placental insufficiency

 

 

78. A child who has recently been diagnosed with cystic fibrosis is in a pediatric clinic where a nurse is performing an assessment. Which later finding of this disease would the nurse not expect to see at this

time?

A)  Positive sweat test

B)  Bulky greasy stools

C)  Moist, productive cough

D)  Meconium ileus

The correct answer is C:Moist, productive cough

 

 

79. Which of the following should the nurse implement to prepare a client for a KUB (Kidney, Ureter, Bladder) radiograph test?

A)  Client must be NPO before the examination

B)  Enema to be administered prior to the examination

C)  Medicate client with Lasix 20 mg IV 30 minutes prior to the examination

D)  No special orders are necessary for this examination

The correct answer is D: No special orders are necessary for this examination

 

 

80. The hospital has sounded the call for a disaster drill on the evening shift. Which of these clients would the nurse put first on the list to be discharged in order to make a room available for a new admission?

A)     A middle aged client with a history of being ventilator dependent for over 7 years and admitted with bacterial pneumonia five days ago

B)     A young adult with diabetes mellitus Type 2 for over 10 years and admitted with antibiotic induced diarrhea 24 hours ago

C)     An elderly client with a history of hypertension, hypercholesterolemia and lupus, and was admitted with Stevens- Johnson syndrome that morning

D)     An adolescent with a positive HIV test and admitted for acute cellulitus of the lower leg 48 hours ago

The correct answer is A: A middle aged client with a history of being ventilator dependent for over 7 years and admitted with bacterial pneumonia five days ago

 

 

81. A triage nurse has these 4 clients arrive in the emergency department within 15 minutes. Which client should the triage nurse send back to be seen first?

A) A 2 month old infant with a history of rolling off the bed and has bulging fontanels with crying

B)  A teenager who got a singed beard while camping

C)  An elderly client with complaints of frequent liquid brown colored stools

D) A middle aged client with intermittent pain behind the right scapula

The correct answer is B: A teenager who got singed a singed beard while camping

 

 

82. A client is receiving digoxin (Lanoxin) 0.25 mg. Daily. The health care provider has written a new order to give metoprolol (Lopressor) 25 mg. B.I.D. In assessing the client prior to administering the medications, which of the following should the nurse report immediately to the health care provider?

A)  Blood pressure 94/60

B)  Heart rate 76

C)  Urine output 50 ml/hour

D)  Respiratory rate 16

The correct answer is A: Blood pressure 94/60

 

 

83. A nurse enters a client’s room to discover that the client has no pulse or respirations.

After calling for help, the first action the nurse should take is A) Start a peripheral IV

B)  Initiate closed-chest massage

C)  Establish an airway

D)  Obtain the crash cart

The correct answer is C: Establish an airway

 

 

84. A 3 year-old child comes to the pediatric clinic after the sudden onset of findings that include irritability, thick muffled voice, croaking on inspiration, hot to touch, sit leaning forward, tongue protruding, drooling and suprasternal retractions. What should the nurse do first?

A)  Prepare the child for x-ray of upper airways

B)  Examine the child’s throat

C)  Collect a sputum specimen

D)  Notify the healthcare provider of the child’s status

The correct answer is D: Notify the health care provider of the child'’s status

 

 

85. A nurse is to administer meperidine hydrochloride (Demerol) 100 mg, atropine sulfate (Atropisol) 0.4 mg, and promethizine hydrochloride (Phenergan) 50 mg IM to a pre- operative client. Which action should the nurse take first? A) Raise the side rails on the bed

B)  Place the call bell within reach

C)  Instruct the client to remain in bed

D)  Have the client empty bladder

The correct answer is D: Have the client empty bladder

 

 

86. In children suspected to have a diagnosis of diabetes, which one of the following complaints would be most likely to prompt parents to take their school age child for evaluation? A) Polyphagia

B)  Dehydration

C)  Bed wetting

D)  Weight loss

The correct answer is C: Bed wetting

 

 

87. A client has been newly diagnosed with hypothyroidism and will take levothyroxine (Synthroid) 50 mcg/day by mouth. As part of the teaching plan, the nurse emphasizes that this medication:

A) Should be taken in the morning B) May decrease the client’s energy level

C)  Must be stored in a dark container

D)  Will decrease the client’s heart rate

The correct answer is A: Should be taken in the morning

 

 

88. A client has a Swan-Ganz catheter in place. The nurse understands that this is intended to measure A) Right heart function

B)  Left heart function

C)  Renal tubule function

D)  Carotid artery function

The correct answer is B: Left heart function

 

 

89. Which of these findings indicate that a pump to deliver a basal rate of 10 ml per hour plus PRN for pain break through for morphine drip is not working? A) The client complains of discomfort at the IV insertion site B) The client states “I just can’t get relief from my pain.”

C)  The level of drug is 100 ml at 8 AM and is 80 ml at noon

D)  The level of the drug is 100 ml at 8 AM and is 50 ml at noon

The correct answer is C: The level of drug is 100 ml at 8 AM and is 80 ml at noon

 

90. The nurse is performing a neurological assessment on a client post right CVA. Which finding, if observed by the nurse, would warrant immediate attention?

A)  Decrease in level of consciousness

B)  Loss of bladder control

C)  Altered sensation to stimuli D) Emotional lability

The correct answer is A: Decrease in level of consciousness

 

 

91. When teaching a client with coronary artery disease about nutrition, the nurse should emphasize

A)  Eating 3 balanced meals a day

B)  Adding complex carbohydrates C) Avoiding very heavy meals

D) Limiting sodium to 7 gms per day

The correct answer is C: Avoiding very heavy meals

 

 

92. The nurse is speaking at a community meeting about personal responsibility for health promotion. A participant asks about chiropractic treatment for illnesses. What should be the focus of the nurse’s response?

A)  Electrical energy fields

B)  Spinal column manipulation

C)  Mind-body balance

D)  Exercise of joints

The correct answer is B: Spinal column manipulation

 

 

93. The home health nurse visits a male client to provide wound care and finds the client lethargic and confused. His wife states he fell down the stairs 2 hours ago. The nurse should

A)  Place a call to the client’s health care provider for instructions

B)  Send him to the emergency room for evaluation

C)  Reassure the client’s wife that the symptoms are transient

D)  Instruct the client’s wife to call the doctor if his symptoms become worse

The correct answer is B: Send him to the emergency room for evaluation

 

 

94. While assessing a 1 month-old infant, which finding should the nurse report immediately?

A)  Abdominal respirations

B)  Irregular breathing rate

C)  Inspiratory grunt

D)  Increased heart rate with crying

Review Information: The correct answer is C: Inspiratory grunt

 

95. A client with multiple sclerosis plans to begin an exercise program. In addition to discussing the benefits of regular exercise, the nurse should caution the client to avoid activities which

A)  Increase the heart rate

B)  Lead to dehydration

C)  Are considered aerobic

D)  May be competitive

The correct answer is B: Lead to dehydration

 

 

96. The nurse is caring for a client who had a total hip replacement 4 days ago. Which assessment requires the nurse’s immediate attention?

A)  I have bad muscle spasms in my lower leg of the affected extremity.

B)  “I just can’t ‘catch my breath’ over the past few minutes and I think I am in grave danger.”

C)  “I have to use the bedpan to pass my water at least every 1 to 2 hours.” D) “It seems that the pain medication is not working as well today.”

The correct answer is B: “I just can'’t ‘‘catch my breath’’ over the past few minutes and I think I am in grave danger.”

 

97. The nurse is giving discharge teaching to a client 7 days post myocardial infarction. He asks the nurse why he must wait 6 weeks before having sexual intercourse. What is the best response by the nurse to this question?

A)  “You need to regain your strength before attempting such exertion.”

B)  “When you can climb 2 flights of stairs without problems, it is generally safe.” C) “Have a glass of wine to relax you, then you can try to have sex.”

D) “If you can maintain an active walking program, you will have less risk.”

The correct answer is B: “When you can climb 2 flights of stairs without problems, it is generally safe.”

 

98. What would the nurse expect to see while assessing the growth of children during their school age years?

A)  Decreasing amounts of body fat and muscle mass

B)  Little change in body appearance from year to year C) Progressive height increase of 4 inches each year D) Yearly weight gain of about 5.5 pounds per year

The correct answer is D: Yearly weight gain of about 5.5 pounds per year

 

 

99. The nurse is assigned to care for a client who has a leaking intracranial aneurysm. To minimize the risk of rebleeding , the nurse should plan to

A)  Restrict visitors to immediate family

B)  Avoid arousal of the client except for family visits

C)  Keep client’s hips flexed at no less than 90 degrees

D)  Apply a warming blanket for temperatures of 98 degrees Fahrenheit or less The correct answer is A: Restrict visitors to immediate family

 

 

100. The nurse is performing a gestational age assessment on a newborn delivered 2 hours ago. When comparing findings to the Ballard scale, which situation may affect the score?

A)  Birth weight

B)  Racial differences

C)  Fetal distress in labor

D)  Birth trauma

The correct answer is C: Fetal distress in labor

 

 

101. A 4 year-old hospitalized child begins to have a seizure while playing with hard plastic toys in the hallway. Of the following nursing actions, which one should the nurse do first?

A)  Place the child in the nearest bed

B)  Administer IV medication to slow down the seizure C) Place a padded tongue blade in the child’s mouth

D) Remove the child’s toys from the immediate area

The correct answer is D: Remove the child’’s toys from the immediate area

 

 

102. A client asks the nurse to explain the basic ideas of homeopathic medicine. The

response that best explains this approach is that remedies

A)  Destroy organisms causing disease

B)  Maintain fluid balance

C)  Boost the immune system

D)  Increase bodily energy

The correct answer is C: Boost the immune system

 

103. The nurse is caring for a 2 year-old who is being treated with chelation therapy, calcium disodium edetate, for lead poisoning. The nurse should be alert for which of the following side effects?

A)  Neurotoxicity

B)  Hepatomegaly

C)  Nephrotoxicity

D)  Ototoxicity

The correct answer is C: Nephrotoxicity

 

 

104. The nurse is caring for a 1 year-old child who has 6 teeth. What is the best way for the nurse to give mouth care to this child?

A)  Using a moist soft brush or cloth to clean teeth and gums

B)  Swabbing teeth and gums with flavored mouthwash

C)  Offering a bottle of water for the child to drink

D)  Brushing with toothpaste and flossing each tooth

The correct answer is A: Using a moist soft brush or cloth to clean teeth and gums

 

 

105. At a senior citizens meeting a nurse talks with a client who has diabetes mellitus Type 1. Which statement by the client during the conversation is most predictive of a potential for impaired skin integrity?

A)  “I give my insulin to myself in my thighs.”

B)  “Sometimes when I put my shoes on I don’t know where my toes are.”

C)  “Here are my up and down glucose readings that I wrote on my calendar.” D) “If I bathe more than once a week my skin feels too dry.”

The correct answer is B: “Sometimes when I put my shoes on I don'’t know where my toes are.”

 

 

106. A couple trying to conceive asks the nurse when ovulation occurs. The woman reports a regular 32 day cycle. Which response by the nurse is correct?

A)  Days 7-10

B)  Days 10-13

C)  Days 14-16

D)  Days 17-19

The correct answer is D: Days 17-19

 

 

107. Included in teaching the client with tuberculosis taking INH about follow-up home care, the nurse should emphasize that a laboratory appointment for which of the following lab tests is critical?

A)  Liver function

B)  Kidney function

C)  Blood sugar

D)  Cardiac enzymes

The correct answer is A: Liver function

 

 

108. A 78 year-old client with pneumonia has a productive cough but is confused. Safety protective devices (restraints) have been ordered for this client. How can the nurse prevent aspiration?

A)  Suction the client frequently while restrained

B)  Secure all 4 restraints to 1 side of bed

C)  Obtain a sitter for the client while restrained

D)  Request an order for a cough suppressant

The correct answer is C: Obtain a sitter for the client while restrained

 

 

109. A client with a fractured femur has been in Russell’s traction for 24 hours. Which nursing action is associated with this therapy?

A)  Check the skin on the sacrum for breakdown

B)  Inspect the pin site for signs of infection

C)  Auscultate the lungs for atelectasis

D)  Perform a neurovascular check for circulation

The correct answer is D: Perform a neurovascular check for circulation

 

110. The nurse is caring for a client with extracellular fluid volume deficit. Which of the following assessments would the nurse anticipate finding?

A)  Bounding pulse

B)  Rapid respirations

C)  Oliguria

D)  Neck veins are distended

The correct answer is C: Oliguria

 

 

111. When suctioning a client’s tracheostomy, the nurse should instill saline in order to A) Decrease the client’s discomfort

B)  Reduce viscosity of secretions

C)  Prevent client aspiration

D)  Remove a mucus plug

The correct answer is D: Remove a mucus plug

 

112. A woman in her third trimester complains of severe heartburn. What is appropriate teaching by the nurse to help the woman alleviate these symptoms? A) Drink small amounts of liquids frequently

B)  Eat the evening meal just before retiring

C)  Take sodium bicarbonate after each meal

D)  Sleep with head propped on several pillows

The correct answer is D: Sleep with head propped on several pillows

 

 

113. A nurse is caring for a client who had a closed reduction of a fractured right wrist followed by the application of a fiberglass cast 12 hours ago. Which finding requires the nurse’s immediate attention?

A)  Capillary refill of fingers on right hand is 3 seconds

B)  Skin warm to touch and normally colored C) Client reports prickling sensation in the right hand D) Slight swelling of fingers of right hand

The correct answer is C: Client reports prickling sensation in the right hand

 

114. A newborn is having difficulty maintaining a temperature above 98 degrees Fahrenheit and has been placed in a warming isolate. Which action is a nursing priority?

A)  Protect the eyes of the neonate from the heat lamp

B)  Monitor the neonate’s temperature

C)  Warm all medications and liquids before giving

D)  Avoid touching the neonate with cold hands The correct answer is B: Monitor the neonate’s temperature

 

 

115. The nurse is caring for a client with a myocardial infarction. Which finding requires the nurse’s immediate action?

A) Periorbital edema B) Dizziness spells

C)  Lethargy

D)  Shortness of breath

The correct answer is B: Dizziness spells

 

 

116. A client is admitted with the diagnosis of pulmonary embolism. While taking a history, the client tells the nurse he was admitted for the same thing twice before, the last time just 3 months ago. The

nurse would anticipate the health care provider ordering

A)  Pulmonary embolectomy

B)  Vena caval interruption

C)  Increasing the coumadin therapy to an INR of 3-4

D)  Thrombolytic therapy

The correct answer is B: Vena cava interruption

 

 

117. A 70 year-old woman is evaluated in the emergency department for a wrist fracture of unknown causes. During the process of taking client history, which of these items should the nurse identify as related to the client’s greatest risk factors for osteoporosis?

A)  Menopause at age 50

B)  Has taken high doses of steroids for arthritis for many years

C)  Maintains an inactive lifestyle for the past 10 years

D)  Drinks 2 glasses of red wine each day for the past 30 years

The correct answer is B: Takes steroids for arthritis

 

 

118. Decentralized scheduling is used on a nursing unit. A chief advantage of this management strategy is that it A) Considers client and staff needs

B)  Conserves time for planning

C)  Frees the nurse manager to handle other priorities

D) Allows for requests about special privileges

The correct answer is A: Considers client and staff needs

119. A newborn has hyperbilirubinemia and is undergoing phototherapy with a blanket. Which safety measure is most important during this process? A) Regulate the neonate’s temperature using a radiant heater

B) Withhold feedings while under the phototherapy C) Provide water feedings at least every 2 hours

D) Protect the eyes of neonate from the phototherapy lights

The correct answer is C: Provide water feedings at least every 2 hours

 

 

120. The nurse is at the community center speaking with retired people. To which comment by one of the retirees during a discussion about glaucoma would the nurse give a supportive comment to reinforce correct information?

A)  “I usually avoid driving at night since lights sometimes seem to make things blur.”

B)  “I take half of the usual dose for my sinuses to maintain my blood pressure.”

C)  “I have to sit at the side of the pool with the grandchildren since I can’t swim with this eye problem.”

D)  “I take extra fiber and drink lots of water to avoid getting constipated.” The correct answer is D: “I take extra fiber and drink lots of water to avoid getting constipated.”

 

 

121. On daily cleaning of a tracheostomy, the client coughs and displaces the

tracheostomy tube. The nurse could have avoided this by

A)  placing an obturator at the client’s bedside

B)  having another nurse assist with the procedure

C)  fastening clean tracheostomy ties before removing old ties

D)  Withdraw catheter in a circular motion

The correct answer is C: fastening clean tracheostomy ties before removing old ties

 

 

122. Which contraindication should the nurse assess for prior to giving a child immunizations?

A)  Mild cold symptoms

B)  Chronic asthma

C)  Depressed immune system

D)  Allergy to eggs

The correct answer is C: Depressed immune system

123. The nurse is teaching home care to the parents of a child with acute spasmodic croup. The most important aspect of this care is

A) Sedation as needed to prevent exhaustion B) Antibiotic therapy for 10 to 14 days

C)  Humidified air and increased oral fluids

D) Antihistamines to decrease allergic response

The correct answer is C: Humidified air and increased oral fluids

 

 

124. A newborn delivered at home without a birth attendant is admitted to the hospital for observation. The initial temperature is 35 degrees Celsius (95 degrees Fahrenheit) axillary. The nurse recognizes that cold stress may lead to what complication?

A)  Lowered BMR

B)  Reduced PaO2

C)  Lethargy

D)  Metabolic alkalosis

The correct answer is B: Reduced PaO2

 

 

125. In addition to standard precautions, a nurse should implement contact precautions for which client?

A)  60 year-old with herpes simplex

B)  6 year-old with mononucleosis

C)  45 year-old with pneumonia

D)  3 year-old with scarlet fever

The correct answer is A: 60 year-old with herpes simplex

 

 

126. Which of the following situations is most likely to produce sepsis in the neonate?

A)  Maternal diabetes

B)  Prolonged rupture of membranes

C)  Cesarean delivery

D)  Precipitous vaginal birth

The correct answer is B: Prolonged rupture of membranes

 

 

127. Which client is at highest risk for developing a pressure ulcer?

A)  23 year-old in traction for fractured femur

B)  72 year-old with peripheral vascular disease, who is unable to walk without assistance C) 75 year-old with left sided paresthesia and is incontinent of urine and stool

D) 30 year-old who is comatose following a ruptured aneurysm

The correct answer is C: 75 year-old client with left sided paresthesia and is incontinent

of urine and stool

128. A new nurse manager is responsible for interviewing applicants for a staff nurse position. Which interview strategy would be the best approach?

A)  Vary the interview style for each candidate to learn different techniques

B)  Use simple questions requiring “yes” and “no” answers to gain definitive information * C) Obtain an interview guide from human resources for consistency inn interviewing each candidate

D) Ask personal information of each applicant to assure meeting of job demands The correct answer is C: Obtain an interview guide from human resources for consistency in interviewing each candidate

 

129. A client who is 12 hour post-op becomes confused and says: “Giant sharks are swimming across the ceiling.” Which assessment is necessary to adequately identify the source of this client’s behavior?

A)  Cardiac rhythm strip

B)  Pupillary response

C)  Pulse oximetry

D)  Peripheral glucose stick

The correct answer is C: Pulse oximetry

 

 

130. A client returns from surgery after an open reduction of a femur fracture. There is a small bloodstain on the cast. Four hours later, the nurse observes that the stain has doubled in size. What is the best action for the nurse to take? A) Call the health care provider

B)  Access the site by cutting a window in the cast

C)  Record the findings in the nurse’s notes only

D)  Outline the spot with a pencil and note the time and date on thecast

The correct answer is D: Outline the spot with a pencil and note the time and date on the cast

 

 

131. A nurse assessing the newborn of a mother with diabetes understands that hypoglycemia is related to what pathophysiological process? A) Disruption of fetal glucose supply

B)  Pancreatic insufficiency

C)  Maternal insulin dependency

D)  Reduced glycogen reserves

The correct answer is A: Disruption of fetal glucose supply

 

 

132. The nurse is teaching a parent about side effects of routine immunizations. Which of the following must be reported immediately?

A)  Irritability

B)  Slight edema at site

C)  Local tenderness

D)  Temperature of 102.5 F

The correct answer is D: Temperature of 102.5 F

 

133. The parents of a toddler ask the nurse how long their child will have to sit in a car seat while in the automobile. What is the nurse’s best response to the parents? A) “Your child must use a care seat until he weighs at least 40 pounds."

B) The child must be 5 years of age to use a regular seat belt. C) “Your child must reach a height of 50 inches to sit in a seat belt." D) “The child can use a regular seat belt when he can sit still."

The correct answer is A: “Your child must use a care seat until he weighs at least 40 pounds."

 

 

134. A 16 year-old boy is admitted for Ewing’s sarcoma of the tibia. In discussing his care with the parents, the nurse understands that the initial treatment most often includes

A)  Amputation just above the tumor

B)  Surgical excision of the mass

C)  Bone marrow graft in the affected leg

D)  Radiation and chemotherapy

The correct answer is D: Radiation and chemotherapy

 

135. A client complains of some discomfort after a below the knee amputation. Which action by the nurse is appropriate to do initially?

A)  Conduct guided imagery or distraction

B)  Ensure that the stump is elevated for the initial day

C)  Wrap the stump snugly in an elastic bandage D) Administer opioid narcotics as ordered

The correct answer is B: Ensure that the stump is elevated for the initial day

 

 

136. What is the best way that parents of pre-schoolers can begin teaching their child about injury prevention?

A)  Set good examples themselves

B)  Protect their child from outside influences

C)  Make sure their child understands all the safety rules

D)  Discuss the consequences of not wearing protective devices

The correct answer is A: Set good examples themselves

137. Which oxygen delivery system would the nurse apply that would provide the highest concentrations of oxygen to the client?

A)  Venturi mask

B)  Partial rebreather mask

C)  Non-rebreather mask

D)  Simple face mask

The correct answer is C: Non-rebreather mask

 

 

138. The nurse is teaching the mother of a 5 month-old about nutrition for her baby. Which statement by the mother indicates the need for further teaching? A) “I’m going to try feeding my baby some rice cereal.” B) “When he wakes at night for a bottle, I feed him.”

C)  “I dip his pacifier in honey so he’ll take it.”

D)  “I keep formula in the refrigerator for 24 hours.”

The correct answer is C: “I dip his pacifier in honey so he'’ll take it.”

 

 

139. The nurse is performing an assessment on a client who is cachectic and has developed an enterocutaneous fistula following surgery to relieve a small bowel obstruction. The client’s total protein level is reported as 4.5. Which of the following would the nurse anticipate?

A) Additional potassium will be given IV

B)  Blood for coagulation studies will be drawn

C)  Total parenteral nutrition (TPN) will be started D) Serum lipase levels will be evaluated

The correct answer is C: Total parenteral nutrition (TPN) will be started

 

 

 

141. A newborn weighed 7 pounds 2 ounces at birth. The nurse assesses the newborn at home 2 days later and finds the weight to be 6 pounds 7 ounces. What should the nurse tell the parents about this

weight loss?

A)  The newborn needs additional assessments

B)  The mother should breast feed more often C) A change to formula is indicated D) The loss is within normal limits

The correct answer is D: The loss is within normal limits

 

  

142. During a situation of pain management, which statement is a priority to consider for the ethical guidelines of the nurse?

A) The client’s self-report is the most important consideration B) Cultural sensitivity is fundamental to pain management

C)  Clients have the right to have their pain relieved

D)  Nurses should not prejudge a client’s pain using their own values

The correct answer is A: The client'’s self report is the most important consideration

 

 

143. A 35-year-old client of Puerto Rican-American descent is diagnosed with ovarian cancer. The client states “I refuse both radiation and chemotherapy because they are ‘hot.'” The next action for the nurse to take is to

A)  Document the situation in the notes

B)  Report the situation to the health care provider

C)  Talk with the client’s family about the situation

D)  Ask the client to talk about the concerns about the “hot"treatments

The correct answer is D: Ask the client to talk about the concerns about the “hot” treatments

 

 

144. Which of the following drugs should the nurse anticipate administering to a client before they are to receive electroconvulsive therapy?

A)  Benzodiazepines

B)  Chlorpromazine (Thorazine)

* C) Succinylcholine (Anectine)

D) Thiopental sodium (Pentothal Sodium)

The correct answer is C: Succinylcholine (Anectine)

 

 

145. A client is brought to the emergency room following a motor vehicle accident. When assessing the client one-half hour after admission, the nurse notes several physical changes. Which changes would require the nurse’s immediate attention? A) Increased restlessness

B)  Tachycardia

C)  Tracheal deviation

D)  Tachypnea

The correct answer is C: Tracheal deviation

 

 

146. Which approach is a priority for the nurse who works with clients from many

different cultures?

A)  Speak at least 2 other languages of clients in the neighborhood

B)  Learn about the cultures of clients who are most often encountered

C)  Have a list of persons for referral when interaction with these clients occur D) Recognize personal attitudes about cultural differences and real or expected biases The correct answer is D: Recognize personal attitudes about cultural differences and real or expected biases

 

 

147. A client with chronic obstructive pulmonary disease (COPD) and a history of coronary artery disease is receiving Aminophylline, 25mg/hour. Which one of the following findings by the nurse would require immediate intervention? A) Decreased blood pressure and respirations.

B)  Flushing and headache.

C)  Restlessness and palpitations.

D)  Increased heart rate and blood pressure.

The correct answer is C: Restlessness and palpitations.

 

 

148. The nurse is planning care for an 8 year-old child. Which of the following should be included in the plan of care?

A)  Encourage child to engage in activities in the playroom

B)  Promote independence in activities of daily living

C)  Talk with the child and allow him to express his opinions D) Provide frequent reassurance and cuddling

The correct answer is A: Encourage child to engage in activities in the playroom

.  

149. A pregnant client who is at 34 weeks gestation is diagnosed with a pulmonary embolism (PE). Which of these mediations would the nurse anticipate the health care provider ordering?

A)  Oral Coumadin therapy

B)  Heparin 5000 units subcutaneously b.i.d.

C)  Heparin infusion to maintain the PTT at 1.5-2.5 times the control value

D)  Heparin by subcutaneous injection to maintain the PTT at 1.5 times the control value The correct answer is D: Heparin by subcutaneous injection to maintain the PTT at 1.5 times the control value

 

 

150. The nurse is caring for a client with Hodgkin’s disease who will be receiving radiation therapy. The nurse recognizes that, as a result of the radiation therapy, the client is most likely to experience

A)  High fever

B)  Nausea

C)  Face and neck edema

D)  Night sweats

The correct answer is B: Nausea

 

 

151. While assessing the vital signs in children, the nurse should know that the apical heart rate is preferred until the radial pulse can be accurately assessed at about what age? A) 1 year of age

B)  2 years of age

C)  3 years of age

D)  4 years of age

The correct answer is B: 2 years of age

 

152. Which of these clients, who all have the findings of a board-like abdomen, would the nurse suggest that the health care provider examine first?

A) An elderly client who stated that “My awful pain in my right side suddenly stopped about 3 hours ago.”

B)  A pregnant woman of 8 weeks newly diagnosed with an ectopic pregnancy

C)  A middle-aged client admitted with diverticulitis and has taken only clear liquids for the past week

D) A teenager with a history of falling off a bicycle and did not hit the handle bars The correct answer is A: An elderly client who stated that “My awful pain in my right side suddenly stopped about 3 hours ago.”

 

 

153. A client with a panic disorder has a new prescription for Xanax (Alprazolam). In teaching the client about the drug’s actions and side effects, which of the following should the nurse emphasize? A) Short-term relief can be expected

B)  The medication acts as a stimulant

C)  Dosage will be increased as tolerated

D)  Initial side effects often continue

The correct answer is A: Short-term relief can be expected

 

 

154. Which of these questions is priority when assessing a client with hypertension?

A)  “What over-the-counter medications do you take?”

B)  “Describe your usual exercise and activity patterns.”

C)  “Tell me about your usual diet.”

D)  “Describe your family’s cardiovascular history.”

The correct answer is A: “What over-the-counter medications do you take?”

 

 

155. During a routine check-up, an insulin-dependent diabetic has his glycosylated hemoglobin checked. The results indicate a level of 11%. Based on this result, what teaching should the nurse emphasize?

A)  Rotation of injection sites

B)  Insulin mixing and preparation

C)  Daily blood sugar monitoring

D)  Regular high protein diet

The correct answer is C: Daily blood sugar monitoring

 

 

156. Which of these clients would the nurse monitor for the complication of C. difficile diarrhea?

A) An adolescent taking medications for acne

B)  An elderly client living in a retirement center taking prednisone

C)  A young adult at home taking a prescribed amino glycoside

D) A hospitalized middle aged client receiving clindamycin

The correct answer is D: A hospitalized middle aged client receiving clindamycin

 

 

157. The nurse is assessing a comatose client receiving gastric tube feedings. Which of the following assessments requires an immediate response from the nurse? A) Decreased breath sounds in right lower lobe

B)  Aspiration of a residual of 100cc of formula

C)  Decrease in bowel sounds

D)  Urine output of 250 cc in past 8 hours

The correct answer is A: Decreased breath sounds in right lower lobe

 

158. The nurse is preparing to take a toddler’s blood pressure for the first time. Which of the following actions should the nurse do first? A) Explain that the procedure will help him to get well

B)  Show a cartoon character with a blood pressure cuff

C)  Explain that the blood pressure checks the heart pump

D)  Permit handling the equipment before putting the cuff in place

The correct answer is D: Permit handling the equipment before putting the cuff in place

 

 

159. A 72 year-old client is scheduled to have a cardioversion. A nurse reviews the client’s medication administration record. The nurse should notify the health care

provider if the client received which medication during the preceding 24 hours?

A)  digoxin (Lanoxin)

B)   diltiazam (Cardizem)

C)   nitroglycerine ointment D) metoprolol (Toprol XL)

The correct answer is A: digoxin (Lanoxin)

 

160. To prevent drug resistance common to tubercle bacilli, the nurse is aware that which of the following agents are usually added to drug therapy?

A)  Anti-inflammatory agent

B)  High doses of B complex vitamins

C)  Amino glycoside antibiotic

D)  Two anti-tuberculosis drugs

The correct answer is D: Two anti-tuberculosis drugs